ACLS Cardio Flashcards

1
Q

What is this rhythm?

A

Second-degree atrioventricular block (Mobitz I Wenckebach)

How well did you know this?
1
Not at all
2
3
4
5
Perfectly
2
Q

What is this rhythm?

A

SVT

How well did you know this?
1
Not at all
2
3
4
5
Perfectly
3
Q

What is this rhythm?

A

Second-degree atrioventricular block (Mobitz Il block)

How well did you know this?
1
Not at all
2
3
4
5
Perfectly
4
Q

What is this rhythm?

A

Monomorphic ventricular tachycardia

How well did you know this?
1
Not at all
2
3
4
5
Perfectly
5
Q

What is this rhythm?

A

V Fib

How well did you know this?
1
Not at all
2
3
4
5
Perfectly
6
Q

What is this rhythm?

A

Second-degree atrioventricular block (Mobitz I Wenckebach)

How well did you know this?
1
Not at all
2
3
4
5
Perfectly
7
Q

What is this rhythm?

A

A Fib

How well did you know this?
1
Not at all
2
3
4
5
Perfectly
8
Q

What is this rhythm?

A

V Fib

How well did you know this?
1
Not at all
2
3
4
5
Perfectly
9
Q

What is this rhythm?

A

Polymorphic ventricular tachycardia

How well did you know this?
1
Not at all
2
3
4
5
Perfectly
10
Q

What is this rhythm?

A

Sinus Bradycardia

How well did you know this?
1
Not at all
2
3
4
5
Perfectly
11
Q

What is this rhythm?

A

Atrial flutter

How well did you know this?
1
Not at all
2
3
4
5
Perfectly
12
Q

What is this rhythm?

A

PEA

How well did you know this?
1
Not at all
2
3
4
5
Perfectly
13
Q

What is this rhythm?

A

Sinus Bradycardia

How well did you know this?
1
Not at all
2
3
4
5
Perfectly
14
Q

What is this rhythm?

A

Sinus Tachycardia

How well did you know this?
1
Not at all
2
3
4
5
Perfectly
15
Q

What is this rhythm?

A

Third-degree atrioventricular block

How well did you know this?
1
Not at all
2
3
4
5
Perfectly
16
Q

What is this rhythm?

A

V Fib

How well did you know this?
1
Not at all
2
3
4
5
Perfectly
17
Q

A patient is in refractory ventricular fibrillation and has received multiple appropriate defibrillation shocks, epinephrine 1 mg IV twice, and an initial dose of amiodarone 300 mg IV. The patient is intubated. Which best describes the recommended second dose of amiodarone for this patient?

A. 1 mg/kg IV push
B. 300 mg IV push
C. 150 mg IV push
D. 1 to 2 mg/min infusion

A

The recommended second dose of amiodarone for a patient in refractory ventricular fibrillation who has already received an initial dose of 300 mg IV is 150 mg IV push. This is in accordance with Advanced Cardiac Life Support (ACLS) guidelines for the management of ventricular fibrillation resistant to initial treatment.

Therefore, the correct option is:
C. 150 mg IV push

How well did you know this?
1
Not at all
2
3
4
5
Perfectly
18
Q

What is the indication for the use of magnesium in cardiac arrest?

A. Shock-refractory monomorphic ventricular tachycardia
B. Pulseless ventricular tachycardia-associated torsades de pointes
C. Shock-refractory ventricular fibrillation
D. Ventricular tachycardia associated with a normal QT interval

A

Magnesium is indicated for use in cardiac arrest specifically for cases of torsades de pointes, a type of polymorphic ventricular tachycardia that often occurs in the context of a prolonged QT interval. It is used when torsades de pointes is suspected or confirmed, especially if it is causing a pulseless ventricular tachycardia.

Therefore, the correct option is:
B. Pulseless ventricular tachycardia-associated torsades de pointes

19
Q

A patient with sinus bradycardia and a heart rate of 42/min has diaphoresis and a blood pressure of 80/60 mm Hg. What is the initial dose of atropine?

A. 0.5 mg
B. 3 mg
C. 0.1 mg
D. 1 mg

A

D. 1 mg

20
Q

A patient has a rapid irregular wide-complex tachycardia. The ventricular rate is 138/min. He is asymptomatic, with a blood pressure of 110/70 mm
Hg. He has a history of angina. What action is recommended next?

A. Giving adenosine 6 mg IV bolus
B. Giving lidocaine 1 to 1.5 mg IV bolus
C. Performing synchronized cardioversion
D. Seeking expert consultation

A

For a patient presenting with a rapid irregular wide-complex tachycardia, and being asymptomatic with a stable blood pressure, the initial management should be focused on diagnosing the rhythm accurately and understanding the underlying cause. Given the complexity of wide-complex tachycardias and the patient’s history of angina, this could be ventricular tachycardia or a supraventricular tachycardia with aberrancy.

Since the patient is hemodynamically stable and asymptomatic, there is not an immediate need for interventions like adenosine, lidocaine, or synchronized cardioversion, which are more urgent in unstable situations. Instead, the recommended action is to seek expert consultation to determine the most appropriate intervention, especially to distinguish between ventricular and supraventricular origins of the tachycardia and to guide subsequent management.

Therefore, the correct option is:
D. Seeking expert consultation

21
Q

A 62-year-old man suddenly experienced difficulty speaking and left-sided weakness. He meets initial criteria for fibrinolytic therapy, and a CT scan of the brain is ordered. Which best describes the guidelines for antiplatelet and fibrinolytic therapy?

A. Give aspirin 120 mg and clopidogrel 75 mg orally
B. Give heparin if the CT scan is negative for hemorrhage
C. Hold aspirin for at least 24 hours if rtPA is administered
D. Give aspirin 162 to 325 mg to be chewed immediately

A

In the scenario described, where a patient presents with symptoms suggestive of an acute ischemic stroke and is being considered for fibrinolytic therapy (e.g., rtPA, recombinant tissue plasminogen activator), the guidelines recommend careful management of concurrent medications that could increase bleeding risk.

It is specifically recommended that antiplatelet therapy, such as aspirin, should be withheld for at least 24 hours after the administration of rtPA. This is to avoid increasing the risk of bleeding, which is a significant concern when using fibrinolytic therapy. The patient should not receive aspirin or other antiplatelets immediately if fibrinolytic therapy is given, until it is confirmed that there is no bleeding complication post-thrombolysis.

Therefore, the correct option is:
C. Hold aspirin for at least 24 hours if rtPA is administered

22
Q

A 35-year-old woman has palpitations, light-headedness, and a stable tachycardia. The monitor shows a regular narrow-complex QRS at a rate of 180/min. Vagal maneuvers have not been effective in terminating the rhythm. An IV has been established. Which drug should be administered?

A. Epinephrine 2 to 10 mcg/kg per minute
B. Lidocaine 1 mg/kg
C. Atropine 0.5 mg
D. Adenosine 6 mg

A

For a patient exhibiting symptoms like palpitations and light-headedness with a stable, regular narrow-complex tachycardia, where vagal maneuvers have been ineffective, the appropriate drug to administer is adenosine. Adenosine is effective for terminating certain types of supraventricular tachycardia (SVT), which often present with a narrow complex QRS pattern. The initial dose of adenosine recommended is 6 mg given rapidly as an intravenous (IV) bolus, followed by a saline flush.

Therefore, the correct option is:
D. Adenosine 6 mg

23
Q

A patient with possible STEMI has ongoing chest discomfort. What is a contraindication to nitrate administration?

A. Use of a phosphodiesterase inhibitor within the previous 24 hours
B. Heart rate less than 90/min
C. Anterior wall myocardial infarction
D. Systolic blood pressure greater than 180 mm Hg

A

One of the contraindications to the administration of nitrates, such as nitroglycerin, in the setting of acute myocardial infarction (AMI) or chest pain is the recent use of phosphodiesterase inhibitors. Phosphodiesterase inhibitors are commonly used for erectile dysfunction and can cause severe hypotension when combined with nitrates due to their synergistic effect on vascular smooth muscle relaxation.

Therefore, the correct option is:
A. Use of a phosphodiesterase inhibitor within the previous 24 hours

24
Q

A patient is in cardiac arrest. Ventricular fibrillation has been refractory to a second shock. Which drug should be administered first?

A. Sodium bicarbonate 50 mEq IV/IO
B. Lidocaine 1 mg/kg IV/10
C. Epinephrine 1 mg IV/IO
D. Atropine 1 mg IV/IO

A

In the management of cardiac arrest with refractory ventricular fibrillation, epinephrine is the first drug recommended after initial defibrillation attempts have failed. According to Advanced Cardiac Life Support (ACLS) guidelines, epinephrine should be administered at a dose of 1 mg IV/IO, which can be repeated every 3-5 minutes during the resuscitation effort.

Therefore, the correct option is:
C. Epinephrine 1 mg IV/IO

25
Q

A patient is in cardiac arrest. High-quality chest compressions are being given. The patient is intubated, and an IV has been started. The rhythm is asystole. What is the first drug/dose to administer?

A. Epinephrine 1 mg IV/IO
B. Atropine 1 mg V/lo
C. Atropine 0.5 mg IV/IO
D. Dopamine 2 to 20 mcg/kg per minute IV/IO

A

In the case of cardiac arrest where the rhythm is asystole, the first drug and dose to administer according to current Advanced Cardiac Life Support (ACLS) guidelines is epinephrine. The recommended dose is 1 mg administered intravenously or intraosseously (IV/IO), and this can be repeated every 3-5 minutes during resuscitation.

Therefore, the correct option is:
A. Epinephrine 1 mg IV/IO

26
Q

A patient is in pulseless ventricular tachycardia. Two shocks and 1 dose of epinephrine have been given. Which drug should be given next?

A. Amiodarone 300 mg
B. Epinephrine 3 mg
C. Adenosine 6 mg
D. Lidocaine 0.5 mg/kg

A

For a patient in pulseless ventricular tachycardia who has already received two shocks and one dose of epinephrine, the next recommended drug according to Advanced Cardiac Life Support (ACLS) guidelines is amiodarone. The initial dose of amiodarone for this scenario is 300 mg IV/IO.

Therefore, the correct option is:
A. Amiodarone 300 mg

27
Q

You arrive on the scene with the code team. High-quality CPR is in progress. An AED has previously advised “no shock indicated.” A rhythm check now finds asystole. After resuming high-quality compressions, which action do you take next?
Choose the correct option and select submit.
A. Establish IV or 10 access
B. Insert a laryngeal airway
C. Perform endotracheal intubation
D. Call for a pulse check

A

In the situation described where high-quality CPR is ongoing and the AED has previously advised “no shock indicated” with a rhythm check revealing asystole, the next step after resuming compressions is to ensure that there is a secure route for medication administration and advanced airway management if needed. Establishing intravenous (IV) or intraosseous (IO) access is crucial for administering necessary medications such as epinephrine during cardiac arrest management.

Therefore, the correct option is:
A. Establish IV or IO access

28
Q

A patient is in refractory ventricular fibrillation. High-quality CPR is in progress. One dose of epinephrine was given after the second shock. An antiarrhythmic drug was given immediately after the third shock. You are the team leader. Which medication do you order next?

A. Epinephrine 1 mg
B. Sodium bicarbonate 50 mEq
C. A second dose of the antiarrhythmic drug
D. Epinephrine 3 mg

A

In the scenario of refractory ventricular fibrillation where high-quality CPR is in progress, and one dose of epinephrine and an antiarrhythmic drug have already been administered following the appropriate shocks, the next step in the Advanced Cardiac Life Support (ACLS) algorithm would be to continue with the administration of epinephrine every 3-5 minutes. Since epinephrine was given after the second shock, and assuming sufficient time has passed or an additional shock has been given without return of spontaneous circulation, the next appropriate action would be to administer another dose of epinephrine.

Therefore, the correct option is:
A. Epinephrine 1 mg

29
Q

A patient with STEMI has ongoing chest discomfort. Heparin 4000 units IV bolus and a heparin infusion of 1000 units per hour are being administered. The patient did not take aspirin because he has a history of gastritis, which was treated 5 years ago. What is your next action?

A. Give clopidogrel 300 mg orally
B. Give enteric-coated aspirin 325 mg rectally
C. Give enteric-coated aspirin 75 mg orally
D. Give aspirin 162 to 325 mg to chew

A

In the management of STEMI, aspirin is a cornerstone treatment because it rapidly inhibits platelet aggregation and reduces the risk of thrombus formation, which is vital in the acute phase of myocardial infarction. Even though the patient has a history of gastritis, the urgent need to manage the STEMI outweighs the potential risks associated with aspirin intake, especially since the gastritis was treated 5 years ago. The best form of aspirin to administer in this scenario to ensure rapid onset of action is to have the patient chew non-enteric coated aspirin.

Therefore, the correct option is:
D. Give aspirin 162 to 325 mg to chew

30
Q

A monitored patient in the ICU developed a sudden onset of narrow-complex tachycardia at a rate of 220/min. The patient’s blood pressure is 128/58 mm Hg, the PETCO is 38 mm Hg, and the pulse oximetry reading is 98%. There is vascular access in the left arm, and the patient has not been given any vasoactive drugs. A 12-lead ECG confirms a supraventricular tachycardia with no evidence of ischemia or infarction. The heart rate has not responded to vagal maneuvers. What is your next action?

A. Administer amiodarone 300 mg V push
B. Perform synchronized cardioversion at 50 J
C. Administer adenosine 6 mg V push
D. Perform synchronized cardioversion at 200 J

A

Given the scenario where the patient in the ICU develops a narrow-complex tachycardia confirmed as supraventricular tachycardia (SVT) on a 12-lead ECG, and vagal maneuvers have failed to terminate the tachycardia, the next step in the management would typically involve pharmacologic intervention. Adenosine is the drug of choice for terminating SVT due to its ability to transiently block the AV node, which can break the reentrant circuit common in SVTs.

The initial recommended dose of adenosine for SVT is 6 mg given rapidly as an IV push, followed by a rapid saline flush. If the initial dose does not convert the rhythm, a subsequent 12 mg dose may be given.

Therefore, the correct option is:
C. Administer adenosine 6 mg IV push

31
Q

A 57-year-old woman has palpitations, chest discomfort, and tachycardia. The monitor shows a regular wide-complex QRS at a rate of 180/min. She becomes diaphoretic, and her blood pressure is 80/60 mm Hg. Which action do you take next?

A. Seek expert consultation
B. Obtain a 12-lead ECG
C. Perform electrical cardioversion
D. Establish V access

A

In the case of a 57-year-old woman who presents with palpitations, chest discomfort, and a regular wide-complex tachycardia at a rate of 180/min, becoming diaphoretic and hypotensive indicates hemodynamic instability. When a patient exhibits signs of instability with a wide-complex tachycardia, which could be ventricular tachycardia or supraventricular tachycardia with aberrancy, immediate action is required to stabilize the patient.

Given her low blood pressure and symptoms, the most appropriate and urgent intervention is electrical cardioversion. This approach is recommended for hemodynamically unstable patients with ventricular tachycardia or other uncertain wide-complex tachycardias.

Therefore, the correct option is:
C. Perform electrical cardioversion

32
Q

A patient has sinus bradycardia with a heart rate of 36/min. Atropine has been administered to a total dose of 3 mg. A transcutaneous pacemaker has failed to capture. The patient is confused, and her blood pressure is 88/56 mm Hg. Which therapy is now indicated?

A. Atropine 1 mg
B. Normal saline 250 mL to 500 mL bolus
C. Epinephrine 2 to 10 mcg/min
D. Adenosine 6 mg

A

In the situation described, where a patient has sinus bradycardia with a significantly low heart rate, atropine has been maximally dosed, and transcutaneous pacing has failed to capture, the patient remains symptomatic and hemodynamically unstable. In this scenario, pharmacological support to increase heart rate and improve hemodynamic status is required.

Given the failure of atropine and pacemaker, the best next step is to administer a continuous infusion of a chronotropic agent such as epinephrine. Epinephrine will help increase the heart rate and can also support blood pressure.

Therefore, the correct option is:
C. Epinephrine 2 to 10 mcg/min

33
Q

You are caring for a 66-year-old man with a history of a large intracerebral hemorrhage 2 months ago. He is being evaluated for another acute stroke. The CT scan is negative for hemorrhage. The patient is receiving oxygen via nasal cannula at 2 L/min, and an IV has been established. His blood pressure is 180/100 mm Hg. Which drug do you anticipate giving to this patient?

A. Nicardipine
B. rtPA
C. Aspirin
D. Glucose (D50)

A

B. Aspirin

In the setting where the patient has a history of a previous intracerebral hemorrhage and a current CT scan that shows no fresh hemorrhage, aspirin can be considered for secondary prevention of ischemic stroke, assuming he’s being evaluated for an ischemic stroke and there are no current indications for thrombolytics like rtPA. Aspirin is commonly used in the management of ischemic stroke for its antiplatelet properties, which help prevent clot formation. This can be crucial after an ischemic stroke is confirmed and there’s no risk of hemorrhage as shown by the CT scan. If the situation involves managing stroke risk post-discharge or in a longer-term setting where acute hemorrhagic risk is not an issue, aspirin is an appropriate and common choice.

34
Q

A patient is in cardiac arrest. Ventricular fibrillation has been refractory to an initial shock. If no pathway for medication administration is in place, which method is preferred?

A. Endotracheal tube
B. Central line
C. IV or 10
D. External jugular vein

A

In the scenario of a cardiac arrest where ventricular fibrillation has been refractory to an initial shock and there is no existing pathway for medication administration, the preferred method to quickly establish a route for drug administration is through an intravenous (IV) or intraosseous (IO) line. These methods provide rapid and reliable access for the administration of emergency medications like epinephrine and amiodarone, which are crucial in the management of cardiac arrest.

C. IV or IO is the correct option because it offers the fastest and most effective means of delivering lifesaving medications directly into the circulation.

35
Q

After initiation of CPR and 1 shock for ventricular fibrillation, this rhythm is present on the next rhythm check. A second shock is given, and chest compressions are resumed immediately. An IV is in place, and no drugs have been given. Bag-mask ventilations are producing visible chest rise.

What is your next intervention?
A. Administer 3 sequential (stacked) shocks at 320 J (monophasic defibrillator)
B. Give amiodarone 300 mg IV/IO
C. Intubate and administer 100% oxygen
D. Give epinephrine 1 mg IV/IO

A

D. Give epinephrine 1 mg IV/IO

36
Q

A patient has been resuscitated from cardiac arrest. During post-ROSC treatment, the patient becomes unresponsive, with the rhythm shown here.
Which action is indicated next?

A. Perform synchronized cardioversion
B. Give lidocaine 1 to 1.5 mg/kg IV
C. Give an immediate unsynchronized high-energy shock (defibrillation dose)
D. Repeat amiodarone 300 mg IV

A

C. Give an immediate unsynchronized high-energy shock (defibrillation dose)

37
Q

A 35-year-old woman presents with a chief complaint of palpitations. She has no chest discomfort, shortness of breath, or light-headedness. Her blood pressure is 120/78 mm Hg. Which intervention is indicated first?

A. Adenosine 12 mg IV slow push (over 1 to 2 minutes)
B. Vagal maneuvers
C. Metoprolol 5 mg IV and repeat if necessary
D. Adenosine 3 mg IV bolus

A

B. Vagal maneuvers

37
Q

A patient becomes unresponsive. You are uncertain if a faint pulse is present. The rhythm shown here is seen on the cardiac monitor. An IV is in place. Which action do
you take next?

A. Administer atropine 1 mg
B. Begin transcutaneous pacing
C. Administer epinephrine 1 mg IV
D. Start high-quality CPR

A

D. Start high-quality CPR

37
Q

You arrive on the scene to find CPR in progress. Nursing staff report the patient was recovering from a pulmonary embolism and suddenly collapsed. Two shocks have been delivered, and an IV has been initiated. What do you administer now?

A. Endotracheal intubation
B. Atropine 0.5 mg IV
C. Epinephrine 1 mg IV
D. Transcutaneous pacing

A

C. Epinephrine 1 mg IV

38
Q

After providing a shock, the provider should:

A. Resume chest compressions
B. Prepare to deliver a second shock
C. Check the pulse rate
D. Start rescue breathing

A

A. Resume chest compressions

39
Q

A patient was in refractory ventricular fibrillation. A third shock has just been administered. Your team looks to you for instructions. What is your next action?

A. Give atropine 1 mg IV
B. Give amiodarone 300 mg IV
C. Check the carotid pulse
D. Resume high-quality chest compressions

A

D. Resume high-quality chest compressions

40
Q

A patient’s 12-lead ECG is transmitted by the paramedics and shows a STEMI. When the patient arrives in the emergency department, the rhythm shown here is seen on the cardiac monitor. The patient has resolution of moderate (5/10) chest pain after 3 doses of sublingual nitroglycerin. Blood pressure is 104/70 mm Hg. Which intervention is most important in reducing this patient’s in-hospital and 30-day mortality rate?

A. Reperfusion therapy
B. Nitroglycerin administration
C. Atropine administration
D. Application of transcutaneous pacemaker

A

A. Reperfusion therapy

41
Q
A